({{Template:Author|Arjun Suresh|{{arjunweb}} }})
 
(18 intermediate revisions by the same user not shown)
Line 5: Line 5:
 
$L_1 = \{< M, w, 1 > |$ $M$ does not halts on $w\}$  
 
$L_1 = \{< M, w, 1 > |$ $M$ does not halts on $w\}$  
  
Here $< M, w, i >$ is a triplet, whose first component. $M$ is an encoding of a Turing  
+
Here $< M, w, i >$ is a triplet, whose first component $M$ is an encoding of a Turing  
Machine, second component,$ w$, is a string, and third component, $i$, is a bit.  
+
Machine, second component $ w$ is a string, and third component $i$ is a bit.  
 +
 
 
Let $L = L_0 ∪ L_1$. Which of the following is true ?  
 
Let $L = L_0 ∪ L_1$. Which of the following is true ?  
 
   
 
   
Line 17: Line 18:
 
'''(D) Neither $L$ nor $L'$ is recursively enumerable '''
 
'''(D) Neither $L$ nor $L'$ is recursively enumerable '''
  
===Solution===
+
==={{Template:Author|Arjun Suresh|{{arjunweb}} }}===
 +
 
 +
Both $L$ and $Lʼ$ are undecidable and not even semi-decidable (not recursively-enumerable). Because halting problem can be solved with both $L$ and $Lʼ$.
 +
 
 +
Halting problem can be stated as follows: A machine $M$ and a word $w$ are given. You have to tell, if $M$ halts on $w$.
 +
 +
So, to solve halting problem $<M,w>$ using $L$, just give $<M,w,0>$ and $<M,w,1>$ to two instances of $T$ which is the Turing machine for $L$. If $T$ accepts the triplet $<M,w,0>$, it means $M$ halts on $w$ => we have solved halting problem. If $T$ accepts the triplet $<M,w,1>$, it means $M$ doesn't halt on $w$ => we have solved halting problem. We know that either $<M,w,0>$ or $<M,w,1>$ is in $L$. So, if $L$ is recursively enumerable, $T$ is bound to stop on at least one of these inputs ($TM$ for a recursively enumerable language stops and accepts, when provided with a word in its language). 
  
Both $L$ and $L'$ are undecidable. Because halting problem can be solved with both $L$ and $L'$.
+
Hence, using $L$ we can solve halting problem => $L$ is not recursively enumerable.
Halting problem can be stated as follows: A machine $M$ and a word $w$ are given. You have to tell, if $M$ halts on $w$.
+
Similarly, we can also show that halting problem can be solved with $$.  
So, to solve halting problem $<M,w>$ using $L$, just give $<M,w,0>$ and $<M,w,1>$ to two instances of $T$ which is the Turing machine for $L$. If $T$ accepts the triplet $<M,w,0>$, it means $M$ halts on $w$ => we have solved halting problem. If $T$ accepts the triplet $<M,w,1>$, it means $M$ doesn't halt on $w$ => we have solved halting problem. We know that either $<M,w,0>$ or $<M,w,1>$ is in $L$. So, if $L$ is recursively enumerable, $T$ is bound to stop on at least one of these inputs (<math>TM</math> for a recursively enumerable language stops and accepts, when provided with a word in its language).  Hence, using $L$ we can solve halting problem       => $L$ is not recursively enumerable.
 
Similarly, we can also show that halting problem can be solved with $L'$.
 
Hence, neither $L$ nor $L'$ is recursively enumerable.
 
  
 +
Hence, neither $L$ nor $Lʼ$ is recursively enumerable.
 +
<!--
 
===Alternate Solution===
 
===Alternate Solution===
  
$L_0$ is recursively enumerable. (Given $<M,w,0>$, we can just give $w$ to $M$. If $M$ halts on $w$, $<M,w,0>$ is element of $L_0$.  
+
$L_0$ is recursively enumerable. (Given $<M,w,0>$, we can just give $w$ to $M$. If $M$ halts on $w$, $<M,w,0>$ is an element of $L_0$.)
$L_1$ is not recursively enumerable. Because, halting problem can be solved with it. To decide if a turing machine $M$ accepts a word $w$, just give $<M,w,1>$ to the Turing machine for $L_1$ and also give $w$ to $M$. Either $M$ accepts $w$, or the Turing machine for $L_1$ accepts $<M,w,1>$. In either case we have solved halting problem. Hence, $L_1$ is not recursively enumerable.
+
 
$L_1$ can be reduced to $L_0'$, and hence $L_0'$ also is not recursively enumerable.
+
$L_1$ is not recursively enumerable. Because, halting problem can be solved with it. To decide if a Turing machine $M$ accepts a word $w$, just give $<M,w,1>$ to the Turing machine for $L_1$ and also give $w$ to $M$. Either $M$ accepts $w$, or the Turing machine for $L_1$ accepts $<M,w,1>$. In either case we have solved halting problem. Hence, $L_1$ is not recursively enumerable.
$L_1'$ can be reduced to $L_0$, and hence $L_1'$ is recursively enumerable.  
+
 
Now, $L$ = $L_0 U L_1$
+
$L_1$ can be reduced to $L_0'$, and hence $L_0'$ also is not recursively enumerable.
= <math>RE</math> U not <math>RE</math>
+
 
= not <math>RE</math>
+
$L_1'$ can be reduced to $L_0$, and hence $L_1'$ is recursively enumerable.  
$L' = (L_0 \cup L_1)'$
+
 
$= L_0' ∩ L_1'$
+
Now, $L$ = $L_0 U L_1$
$=$ not <math>RE</math> $∩$ <math>RE</math>
+
 
$=$ not <math>RE</math>
+
= <math>RE</math> U not <math>RE</math>
 +
 
 +
= not <math>RE</math>
 +
 
 +
$= (L_0 \cup L_1)'$
 +
 
 +
$= L_0' ∩ L_1'$
 +
 
 +
$=$ not <math>RE</math> $∩$ <math>RE</math>
 +
 
 +
$=$ not <math>RE</math>
  
 +
-->
 
{{Template:FBD}}
 
{{Template:FBD}}
  
Line 44: Line 61:
  
 
[[Category: GATE2003]]
 
[[Category: GATE2003]]
[[Category: Automata Theory]]
+
[[Category: Automata questions from GATE]]
[[Category: Previous year GATE questions]]
 

Latest revision as of 19:40, 10 September 2014

Define languages L0 and L1 as follows :

$L_0 = \{< M, w, 0 > |$ $M$ halts on $w\} $

$L_1 = \{< M, w, 1 > |$ $M$ does not halts on $w\}$

Here $< M, w, i >$ is a triplet, whose first component $M$ is an encoding of a Turing Machine, second component $ w$ is a string, and third component $i$ is a bit.

Let $L = L_0 ∪ L_1$. Which of the following is true ?

(A) $L$ is recursively enumerable, but is not

(B) $L$ is recursively enumerable, but $ L'$ is not

(C) Both $L$ and $L'$ are recursive

(D) Neither $L$ nor $L'$ is recursively enumerable

Solution by Arjun Suresh

Both $L$ and $Lʼ$ are undecidable and not even semi-decidable (not recursively-enumerable). Because halting problem can be solved with both $L$ and $Lʼ$.

Halting problem can be stated as follows: A machine $M$ and a word $w$ are given. You have to tell, if $M$ halts on $w$.

So, to solve halting problem $<M,w>$ using $L$, just give $<M,w,0>$ and $<M,w,1>$ to two instances of $T$ which is the Turing machine for $L$. If $T$ accepts the triplet $<M,w,0>$, it means $M$ halts on $w$ => we have solved halting problem. If $T$ accepts the triplet $<M,w,1>$, it means $M$ doesn't halt on $w$ => we have solved halting problem. We know that either $<M,w,0>$ or $<M,w,1>$ is in $L$. So, if $L$ is recursively enumerable, $T$ is bound to stop on at least one of these inputs ($TM$ for a recursively enumerable language stops and accepts, when provided with a word in its language).

Hence, using $L$ we can solve halting problem => $L$ is not recursively enumerable. Similarly, we can also show that halting problem can be solved with $Lʼ$.

Hence, neither $L$ nor $Lʼ$ is recursively enumerable.



blog comments powered by Disqus

Define languages L0 and L1 as follows :

$L_0 = \{< M, w, 0 > |$ $M$ halts on $w\} $

$L_1 = \{< M, w, 1 > |$ $M$ does not halts on $w\}$

Here $< M, w, i >$ is a triplet, whose first component. $M$ is an encoding of a Turing Machine, second component,$ w$, is a string, and third component, $i$, is a bit. Let $L = L_0 ∪ L_1$. Which of the following is true ?

(A) $L$ is recursively enumerable, but is not

(B) $L$ is recursively enumerable, but $ L'$ is not

(C) Both $L$ and $L'$ are recursive

(D) Neither $L$ nor $L'$ is recursively enumerable

Solution[edit]

Both $L$ and $L'$ are undecidable. Because halting problem can be solved with both $L$ and $L'$. 
Halting problem can be stated as follows: A machine $M$ and a word $w$ are given. You have to tell, if $M$ halts on $w$. 
So, to solve halting problem $<M,w>$ using $L$, just give $<M,w,0>$ and $<M,w,1>$ to two instances of $T$ which is the Turing machine for $L$. If $T$ accepts the triplet $<M,w,0>$, it means $M$ halts on $w$ => we have solved halting problem. If $T$ accepts the triplet $<M,w,1>$, it means $M$ doesn't halt on $w$ => we have solved halting problem. We know that either $<M,w,0>$ or $<M,w,1>$ is in $L$. So, if $L$ is recursively enumerable, $T$ is bound to stop on at least one of these inputs (<math>TM</math> for a recursively enumerable language stops and accepts, when provided with a word in its language).  Hence, using $L$ we can solve halting problem        => $L$ is not recursively enumerable.
Similarly, we can also show that halting problem can be solved with $L'$. 
Hence, neither $L$ nor $L'$ is recursively enumerable.

Alternate Solution[edit]

$L_0$ is recursively enumerable. (Given $<M,w,0>$, we can just give $w$ to $M$. If $M$ halts on $w$, $<M,w,0>$ is element of $L_0$. 
$L_1$ is not recursively enumerable. Because, halting problem can be solved with it. To decide if a turing machine $M$ accepts a word $w$, just give $<M,w,1>$ to the Turing machine for $L_1$ and also give $w$ to $M$. Either $M$ accepts $w$, or the Turing machine for $L_1$ accepts $<M,w,1>$. In either case we have solved halting problem. Hence, $L_1$ is not recursively enumerable.
$L_1$ can be reduced to $L_0'$, and hence $L_0'$ also is not recursively enumerable.
$L_1'$ can be reduced to $L_0$, and hence $L_1'$ is recursively enumerable. 
Now, $L$ = $L_0 U L_1$
= <math>RE</math> U not <math>RE</math>
= not <math>RE</math>
$L' = (L_0 \cup L_1)'$
$= L_0' ∩ L_1'$
$=$ not <math>RE</math> $∩$ <math>RE</math>
$=$ not <math>RE</math>




blog comments powered by Disqus